What am I doing wrong? Exercise 2, chapter 2, section 3 from Guillemin and Pollack.












1












$begingroup$


I am doing exercise 2, chapter 2, section 3 from Guillemin and Pollack's ''Differential Topology''. Part of the excercise is to prove that given a compact manifold $Y subset mathbb{R}^m$, and a pont $w in mathbb{R}^m$, there exist a point (not necessarily unique) $y in Y$ closest to $w$. This part I have done, the next part is to prove that $w-y in N_y(Y)$.



($N_y(Y)$ is the orthogonal complement of $T_yY$)



I followed the hint, and since any element in $T_yY$ is the velocity vector of a curve $c$ in $Y$ such that $c(0)=y$, then all i got to prove is that $(w-y) cdot dot{c(0)}=0$ for all of these curves.



Since the function $ g(t) =mid w-c(t) mid^2 = sum_{i=1}^m w_i^2+c(t)_i^2 $ has a minimum at $0$, deriving you get:



$0=g'(0)= 2sum_{i=1}^m c(0)_ic'(0)_i = ycdot dot{c(0)}$, which would mean that $y$ is in $N_y(Y)$. Which is in many cases false.



What is my error here?










share|cite|improve this question











$endgroup$












  • $begingroup$
    The problem is that $w$ and $c(t)$ are not necessarily orthogonal.
    $endgroup$
    – Mauro
    Dec 10 '18 at 19:45
















1












$begingroup$


I am doing exercise 2, chapter 2, section 3 from Guillemin and Pollack's ''Differential Topology''. Part of the excercise is to prove that given a compact manifold $Y subset mathbb{R}^m$, and a pont $w in mathbb{R}^m$, there exist a point (not necessarily unique) $y in Y$ closest to $w$. This part I have done, the next part is to prove that $w-y in N_y(Y)$.



($N_y(Y)$ is the orthogonal complement of $T_yY$)



I followed the hint, and since any element in $T_yY$ is the velocity vector of a curve $c$ in $Y$ such that $c(0)=y$, then all i got to prove is that $(w-y) cdot dot{c(0)}=0$ for all of these curves.



Since the function $ g(t) =mid w-c(t) mid^2 = sum_{i=1}^m w_i^2+c(t)_i^2 $ has a minimum at $0$, deriving you get:



$0=g'(0)= 2sum_{i=1}^m c(0)_ic'(0)_i = ycdot dot{c(0)}$, which would mean that $y$ is in $N_y(Y)$. Which is in many cases false.



What is my error here?










share|cite|improve this question











$endgroup$












  • $begingroup$
    The problem is that $w$ and $c(t)$ are not necessarily orthogonal.
    $endgroup$
    – Mauro
    Dec 10 '18 at 19:45














1












1








1





$begingroup$


I am doing exercise 2, chapter 2, section 3 from Guillemin and Pollack's ''Differential Topology''. Part of the excercise is to prove that given a compact manifold $Y subset mathbb{R}^m$, and a pont $w in mathbb{R}^m$, there exist a point (not necessarily unique) $y in Y$ closest to $w$. This part I have done, the next part is to prove that $w-y in N_y(Y)$.



($N_y(Y)$ is the orthogonal complement of $T_yY$)



I followed the hint, and since any element in $T_yY$ is the velocity vector of a curve $c$ in $Y$ such that $c(0)=y$, then all i got to prove is that $(w-y) cdot dot{c(0)}=0$ for all of these curves.



Since the function $ g(t) =mid w-c(t) mid^2 = sum_{i=1}^m w_i^2+c(t)_i^2 $ has a minimum at $0$, deriving you get:



$0=g'(0)= 2sum_{i=1}^m c(0)_ic'(0)_i = ycdot dot{c(0)}$, which would mean that $y$ is in $N_y(Y)$. Which is in many cases false.



What is my error here?










share|cite|improve this question











$endgroup$




I am doing exercise 2, chapter 2, section 3 from Guillemin and Pollack's ''Differential Topology''. Part of the excercise is to prove that given a compact manifold $Y subset mathbb{R}^m$, and a pont $w in mathbb{R}^m$, there exist a point (not necessarily unique) $y in Y$ closest to $w$. This part I have done, the next part is to prove that $w-y in N_y(Y)$.



($N_y(Y)$ is the orthogonal complement of $T_yY$)



I followed the hint, and since any element in $T_yY$ is the velocity vector of a curve $c$ in $Y$ such that $c(0)=y$, then all i got to prove is that $(w-y) cdot dot{c(0)}=0$ for all of these curves.



Since the function $ g(t) =mid w-c(t) mid^2 = sum_{i=1}^m w_i^2+c(t)_i^2 $ has a minimum at $0$, deriving you get:



$0=g'(0)= 2sum_{i=1}^m c(0)_ic'(0)_i = ycdot dot{c(0)}$, which would mean that $y$ is in $N_y(Y)$. Which is in many cases false.



What is my error here?







differential-topology






share|cite|improve this question















share|cite|improve this question













share|cite|improve this question




share|cite|improve this question








edited Dec 10 '18 at 19:42









Shaun

9,268113684




9,268113684










asked Dec 10 '18 at 19:35









Bajo FondoBajo Fondo

410315




410315












  • $begingroup$
    The problem is that $w$ and $c(t)$ are not necessarily orthogonal.
    $endgroup$
    – Mauro
    Dec 10 '18 at 19:45


















  • $begingroup$
    The problem is that $w$ and $c(t)$ are not necessarily orthogonal.
    $endgroup$
    – Mauro
    Dec 10 '18 at 19:45
















$begingroup$
The problem is that $w$ and $c(t)$ are not necessarily orthogonal.
$endgroup$
– Mauro
Dec 10 '18 at 19:45




$begingroup$
The problem is that $w$ and $c(t)$ are not necessarily orthogonal.
$endgroup$
– Mauro
Dec 10 '18 at 19:45










1 Answer
1






active

oldest

votes


















2












$begingroup$

Note that $displaystylelVert w-c(t)rVert^2=sum_{i=1}^m{w_i}^2-2w_ic_i(t)+{c_i}^2(t)$. It looks like you forgot the middle term.






share|cite|improve this answer









$endgroup$













  • $begingroup$
    Yes! Thank you, and it is a mistake that at this point in my studies I should not be making. The worst part is that I had a feeling it was an error of that sort. I feel ashamed. Thank you.
    $endgroup$
    – Bajo Fondo
    Dec 10 '18 at 19:45










  • $begingroup$
    the site did not let me accept it until 6 minutes had passed from the posting of the question. Thank you again.
    $endgroup$
    – Bajo Fondo
    Dec 10 '18 at 19:56











Your Answer





StackExchange.ifUsing("editor", function () {
return StackExchange.using("mathjaxEditing", function () {
StackExchange.MarkdownEditor.creationCallbacks.add(function (editor, postfix) {
StackExchange.mathjaxEditing.prepareWmdForMathJax(editor, postfix, [["$", "$"], ["\\(","\\)"]]);
});
});
}, "mathjax-editing");

StackExchange.ready(function() {
var channelOptions = {
tags: "".split(" "),
id: "69"
};
initTagRenderer("".split(" "), "".split(" "), channelOptions);

StackExchange.using("externalEditor", function() {
// Have to fire editor after snippets, if snippets enabled
if (StackExchange.settings.snippets.snippetsEnabled) {
StackExchange.using("snippets", function() {
createEditor();
});
}
else {
createEditor();
}
});

function createEditor() {
StackExchange.prepareEditor({
heartbeatType: 'answer',
autoActivateHeartbeat: false,
convertImagesToLinks: true,
noModals: true,
showLowRepImageUploadWarning: true,
reputationToPostImages: 10,
bindNavPrevention: true,
postfix: "",
imageUploader: {
brandingHtml: "Powered by u003ca class="icon-imgur-white" href="https://imgur.com/"u003eu003c/au003e",
contentPolicyHtml: "User contributions licensed under u003ca href="https://creativecommons.org/licenses/by-sa/3.0/"u003ecc by-sa 3.0 with attribution requiredu003c/au003e u003ca href="https://stackoverflow.com/legal/content-policy"u003e(content policy)u003c/au003e",
allowUrls: true
},
noCode: true, onDemand: true,
discardSelector: ".discard-answer"
,immediatelyShowMarkdownHelp:true
});


}
});














draft saved

draft discarded


















StackExchange.ready(
function () {
StackExchange.openid.initPostLogin('.new-post-login', 'https%3a%2f%2fmath.stackexchange.com%2fquestions%2f3034375%2fwhat-am-i-doing-wrong-exercise-2-chapter-2-section-3-from-guillemin-and-polla%23new-answer', 'question_page');
}
);

Post as a guest















Required, but never shown

























1 Answer
1






active

oldest

votes








1 Answer
1






active

oldest

votes









active

oldest

votes






active

oldest

votes









2












$begingroup$

Note that $displaystylelVert w-c(t)rVert^2=sum_{i=1}^m{w_i}^2-2w_ic_i(t)+{c_i}^2(t)$. It looks like you forgot the middle term.






share|cite|improve this answer









$endgroup$













  • $begingroup$
    Yes! Thank you, and it is a mistake that at this point in my studies I should not be making. The worst part is that I had a feeling it was an error of that sort. I feel ashamed. Thank you.
    $endgroup$
    – Bajo Fondo
    Dec 10 '18 at 19:45










  • $begingroup$
    the site did not let me accept it until 6 minutes had passed from the posting of the question. Thank you again.
    $endgroup$
    – Bajo Fondo
    Dec 10 '18 at 19:56
















2












$begingroup$

Note that $displaystylelVert w-c(t)rVert^2=sum_{i=1}^m{w_i}^2-2w_ic_i(t)+{c_i}^2(t)$. It looks like you forgot the middle term.






share|cite|improve this answer









$endgroup$













  • $begingroup$
    Yes! Thank you, and it is a mistake that at this point in my studies I should not be making. The worst part is that I had a feeling it was an error of that sort. I feel ashamed. Thank you.
    $endgroup$
    – Bajo Fondo
    Dec 10 '18 at 19:45










  • $begingroup$
    the site did not let me accept it until 6 minutes had passed from the posting of the question. Thank you again.
    $endgroup$
    – Bajo Fondo
    Dec 10 '18 at 19:56














2












2








2





$begingroup$

Note that $displaystylelVert w-c(t)rVert^2=sum_{i=1}^m{w_i}^2-2w_ic_i(t)+{c_i}^2(t)$. It looks like you forgot the middle term.






share|cite|improve this answer









$endgroup$



Note that $displaystylelVert w-c(t)rVert^2=sum_{i=1}^m{w_i}^2-2w_ic_i(t)+{c_i}^2(t)$. It looks like you forgot the middle term.







share|cite|improve this answer












share|cite|improve this answer



share|cite|improve this answer










answered Dec 10 '18 at 19:43









José Carlos SantosJosé Carlos Santos

163k22130233




163k22130233












  • $begingroup$
    Yes! Thank you, and it is a mistake that at this point in my studies I should not be making. The worst part is that I had a feeling it was an error of that sort. I feel ashamed. Thank you.
    $endgroup$
    – Bajo Fondo
    Dec 10 '18 at 19:45










  • $begingroup$
    the site did not let me accept it until 6 minutes had passed from the posting of the question. Thank you again.
    $endgroup$
    – Bajo Fondo
    Dec 10 '18 at 19:56


















  • $begingroup$
    Yes! Thank you, and it is a mistake that at this point in my studies I should not be making. The worst part is that I had a feeling it was an error of that sort. I feel ashamed. Thank you.
    $endgroup$
    – Bajo Fondo
    Dec 10 '18 at 19:45










  • $begingroup$
    the site did not let me accept it until 6 minutes had passed from the posting of the question. Thank you again.
    $endgroup$
    – Bajo Fondo
    Dec 10 '18 at 19:56
















$begingroup$
Yes! Thank you, and it is a mistake that at this point in my studies I should not be making. The worst part is that I had a feeling it was an error of that sort. I feel ashamed. Thank you.
$endgroup$
– Bajo Fondo
Dec 10 '18 at 19:45




$begingroup$
Yes! Thank you, and it is a mistake that at this point in my studies I should not be making. The worst part is that I had a feeling it was an error of that sort. I feel ashamed. Thank you.
$endgroup$
– Bajo Fondo
Dec 10 '18 at 19:45












$begingroup$
the site did not let me accept it until 6 minutes had passed from the posting of the question. Thank you again.
$endgroup$
– Bajo Fondo
Dec 10 '18 at 19:56




$begingroup$
the site did not let me accept it until 6 minutes had passed from the posting of the question. Thank you again.
$endgroup$
– Bajo Fondo
Dec 10 '18 at 19:56


















draft saved

draft discarded




















































Thanks for contributing an answer to Mathematics Stack Exchange!


  • Please be sure to answer the question. Provide details and share your research!

But avoid



  • Asking for help, clarification, or responding to other answers.

  • Making statements based on opinion; back them up with references or personal experience.


Use MathJax to format equations. MathJax reference.


To learn more, see our tips on writing great answers.




draft saved


draft discarded














StackExchange.ready(
function () {
StackExchange.openid.initPostLogin('.new-post-login', 'https%3a%2f%2fmath.stackexchange.com%2fquestions%2f3034375%2fwhat-am-i-doing-wrong-exercise-2-chapter-2-section-3-from-guillemin-and-polla%23new-answer', 'question_page');
}
);

Post as a guest















Required, but never shown





















































Required, but never shown














Required, but never shown












Required, but never shown







Required, but never shown

































Required, but never shown














Required, but never shown












Required, but never shown







Required, but never shown







Popular posts from this blog

Plaza Victoria

In PowerPoint, is there a keyboard shortcut for bulleted / numbered list?

How to put 3 figures in Latex with 2 figures side by side and 1 below these side by side images but in...